Difference between revisions of "1997 PMWC Problems/Problem T10"

m
Line 4: Line 4:
  
 
==Solution==
 
==Solution==
 
+
Less than 9.
 
{{solution}}
 
{{solution}}
  

Revision as of 06:37, 10 October 2013

Problem

The twelve integers 1, 2, 3,..., 12 are arranged in a circle such that the difference of any two adjacent numbers is either 2, 3 or 4. What is the maximum number of the difference '4' can occur in any such arrangement?

Solution

Less than 9. This problem needs a solution. If you have a solution for it, please help us out by adding it.

See Also

1997 PMWC (Problems)
Preceded by
Problem T9
Followed by
Last
Problem
I: 1 2 3 4 5 6 7 8 9 10 11 12 13 14 15
T: 1 2 3 4 5 6 7 8 9 10